LSAT and Law School Admissions Forum

Get expert LSAT preparation and law school admissions advice from PowerScore Test Preparation.

 Administrator
PowerScore Staff
  • PowerScore Staff
  • Posts: 8917
  • Joined: Feb 02, 2011
|
#23954
Complete Question Explanation

WeakenX-CE, #%. The correct answer choice is (E)

Since this is an Except question as well, we know that among the answer choices there are four that weaken the argument and one that doesn’t.

Answer choice (A): This answer choice does weaken the argument, since it offers an alternative explanation for the increase in spending on cataract surgery. If there are more people to spend on the surgery, that could explain why more is being spent.

Answer choice (B): This answer weakens the argument in the same way, by offering an alternative explanation. If people’s chances of requiring the surgery have increased, this could also explain why more is being spent.

Answer choice (C): This answer choice also offers another possible reason for the increased total spent on the surgery. If there are more old people, there are likely to be more candidates for the surgery. This could be why more money is currently being spent on the operation.

Answer choice (D): If health insurance is more likely to cover the procedure, more people might be likely to have it performed. Making the operation more affordable, this offers an alternative explanation for the increased total expenditures.

Answer choice (E): This is the correct answer choice, as it strengthens the argument in the stimulus. If the risk associated with unsuccessful surgery is greater, that would seem to make it more likely that people would be responsive to the reduced risk of the surgery with the more recent, and safer, technology.
 Tomars
  • Posts: 15
  • Joined: Aug 03, 2017
|
#49486
I am not quite understanding the rationale provided for E. The rationale seems to assume the unsuccessful surgeries were due to the old non-high tech cataract surgery methods.

I am going back and forth on how to understand E, but ultimately it seems to have no effect on the argument. I understand how all other answers would weaken the argument by providing alternate reasons for the increased cost of health care specific to cataract surgery.

At most, E is saying that for some folks who undergo the surgery, they come out with worse eye problems, and it might be reasonable to believe they will need to fix those problems, and essentially spend more doing so. However, we don't know if there is a significant number of those people to substantiate a real increase in the nation's cost of care, so it has no effect on the argument.

At worst, we can't imply anything from what it says... and therefore still has no effect on the argument.
 Adam Tyson
PowerScore Staff
  • PowerScore Staff
  • Posts: 5153
  • Joined: Apr 14, 2011
|
#49733
The reason our explanation of E says that it strengthens the argument is because it suggests that prior to the high-tech advances, patients would have been less likely to elect to have it, or that it was generally considered higher risk and thus only done rarely. The stimulus goes hand in hand with this idea in that it says the improvements in the surgery that made it more effective/successful, which led to more of the procedures being done. That means that when it was less successful, fewer people elected to have it.

That said, there is a good argument to be made that E really has no effect, because it has nothing to do with whether high tech advances are the cause of the increased number of cataract surgeries or not. Your view is valid - nice job! The modern, high-tech surgeries could still occasional be unsuccessful, after all. But even if we take that view that still means E is the best answer, because it does not weaken the argument!

As long as you recognize that E does not weaken the argument, while the other 4 answers do, that is all you need to select it, and no further analysis is required.
 LSAT2020
  • Posts: 31
  • Joined: Jun 24, 2020
|
#76805
I'm struggling to understand how D weakness the argument.

If health insurance is covering more people then it causes cataract surgery to be cheaper for more people. Since it's cheaper, how are we 100% certain that it's driving up the nation's health care cost?

Thank you in advance!
 Frank Peter
PowerScore Staff
  • PowerScore Staff
  • Posts: 99
  • Joined: May 14, 2020
|
#77132
Hi LSAT2020,

This is a cause and effect argument - the author is saying that because cataract surgery has become more effective and less painful, more people are opting to undergo the procedure. The weakness in this argument is that the author hasn't really accounted for potential alternate causes, the most obvious one being changes in cost for the patient. What if cataract surgery was much more expensive previously than it is today, and that was the major deciding factor for patients? (D) would support this counter-argument - if health coverage today more readily covers the surgery than it did previously, that might suggest that it wasn't the pain or effectiveness that was dissuading people from undergoing the surgery - it was the cost.

Get the most out of your LSAT Prep Plus subscription.

Analyze and track your performance with our Testing and Analytics Package.